Riemann Zeta Function showing converges uniformly for s>1

In summary: Then n^s = e^{s log(n)} = e^{(Re(s) + i Im(s)) log(n)}. Then take the modulus of both sides.In summary, the conversation discusses the convergence of the series g(s) = \sum\limits^{\infty}_{n=1} 1/n^{-s} and the use of the Weistrass M test to prove its uniform convergence for Re(s>1). The solution involves finding a suitable M_n and determining the modulus of n^s.
  • #1
binbagsss
1,254
11

Homework Statement



## g(s) = \sum\limits^{\infty}_{n=1} 1/n^{-s}, ##

Show that ##g(s)## converges uniformly for ## Re(s>1) ##

Homework Equations



Okay, so I think the right thing to look at is the Weistrass M test. This tells me that if I can find a ##M_{n}##, a real number, such that for each ##n## , ## | f_{n} | \leq M_{n} ##, and ## \sum\limits^{\infty}_{n=1} M_{n} ## converges, then ## \sum\limits^{\infty}_{n=1} f_{n}(s) ## converges, where ##f_{n}(s)= 1/n^{-s}## here.

The Attempt at a Solution



Okay, so if I consider the real part of ##s## only, it's pretty obvious that such a ##M_{n}## can be found for ##s>1##, i.e. ##M_{n} = 1/n ##.

However I'm pretty stuck on how to incorporate ##Im(s)## into this, which has no bounds specified right?

So say I assume ##Re (s) =1##, and we know that the series is then less than :

##\frac{1}{1^{1+iy}} + \frac{1}{2^{1+iy}} + \frac{1}{1^{3+iy}} + ... ##
= ##\frac{1}{1 . iy} + \frac{1}{2 . iy} + \frac{1}{3 . iy} +... ##

where ##s = 1 + iy ##,

but surely as ##Im(s) -> 0##, the imaginary part of each term in the series blows up, so I'm having a hard time understanding how it is bounded within any contraints on ##Im(s)## and only ##Re(s)##.

Many thanks in advance.
 
Physics news on Phys.org
  • #2
First of all, I think you mean [itex]\frac{1}{n^{+s}}[/itex] not [itex]\frac{1}{n^{-s}}[/itex].

Second, you've got the right idea, that if you can find some [itex]M_n[/itex] such that [itex]|\frac{1}{n^s}| \leq M_n[/itex], and [itex]\sum_n M_n[/itex] converges, then [itex]\sum_n \frac{1}{n^s}[/itex] converges. The easiest choice is to just let [itex]M_n = |\frac{1}{n^s}| [/itex]

Third, you need to figure out what [itex]|\frac{1}{n^s}|[/itex] is. What you wrote is wrong: [itex]2^{1+i} \neq 2 \cdot i[/itex]. Try this: Write [itex]n = e^{log(n)}[/itex], where [itex]log[/itex] means the natural log. And then write [itex]s = Re(s) + i Im(s)[/itex].
 

1. What is the Riemann Zeta Function?

The Riemann Zeta Function is a mathematical function that is defined as the sum of the reciprocal of all positive integers to a given power. It is denoted by the symbol ζ(s), where s is a complex variable.

2. Why is it important to show that the Riemann Zeta Function converges uniformly for s>1?

It is important to show that the Riemann Zeta Function converges uniformly for s>1 because it is a necessary condition for the function to be analytic in that region. This means that the function can be approximated by a polynomial with increasing accuracy, making it easier to study and analyze.

3. What does it mean for the Riemann Zeta Function to converge uniformly?

Convergence uniformity means that the function converges at the same rate in all parts of the domain. In other words, as the value of s increases, the function approaches its limit uniformly across the entire domain of s>1.

4. How is the uniform convergence of the Riemann Zeta Function proven for s>1?

The uniform convergence of the Riemann Zeta Function for s>1 is proven using the Weierstrass M-test. This test states that if a series of functions can be bounded by a convergent series of constants, then the original series also converges uniformly.

5. What implications does the uniform convergence of the Riemann Zeta Function have?

The uniform convergence of the Riemann Zeta Function has important implications in number theory and complex analysis. It allows for the extension of the function to the entire complex plane, which has applications in understanding the distribution of prime numbers and the behavior of other complex functions.

Similar threads

  • Calculus and Beyond Homework Help
Replies
14
Views
527
  • Calculus and Beyond Homework Help
Replies
1
Views
265
  • Calculus and Beyond Homework Help
Replies
1
Views
1K
  • Calculus and Beyond Homework Help
Replies
4
Views
318
  • Calculus and Beyond Homework Help
Replies
3
Views
422
  • Calculus and Beyond Homework Help
Replies
1
Views
1K
  • General Math
Replies
4
Views
1K
  • Calculus and Beyond Homework Help
Replies
6
Views
395
  • Calculus and Beyond Homework Help
Replies
14
Views
1K
  • Topology and Analysis
Replies
3
Views
1K
Back
Top